From: W. Trevor King Date: Wed, 12 Aug 2009 10:59:50 +0000 (-0400) Subject: Added rec8 problems (no solutions yet) X-Git-Url: http://git.tremily.us/?p=course.git;a=commitdiff_plain;h=a117ba7a9208f34764f568a48ca85289b42208fe Added rec8 problems (no solutions yet) --- diff --git a/latex/problems/Young_and_Freedman_12/problem27.39.tex b/latex/problems/Young_and_Freedman_12/problem27.39.tex new file mode 100644 index 0000000..a052690 --- /dev/null +++ b/latex/problems/Young_and_Freedman_12/problem27.39.tex @@ -0,0 +1,61 @@ +\begin{problem*}{27.39} +A thin, $50.0\U{cm}$ long metal bar with mass $750\U{g}$ rests on, but +is not attached to, two metallic supports in a uniform $0.450\U{T}$ +magnetic field, as shown in Fig.~27.51. A battery and a +$25.0\U{\Ohm}$ resistor in series are connected to the +supports. \Part{a} What is the highest voltage the battery can have +without breaking the circuit at the supports? \Part{b} The battery +voltage has the maximum value caculated in \Part{a}. If the resistor +suddenly gets partially short-circuited, decreasing its resistance to +$2.0\U{\Ohm}$, find the initial acceleration of the bar. +\begin{center} +\begin{asy} +import Mechanics; +import ElectroMag; +import Circ; + +real supRadius = 3mm; + +TwoTerminal V = source(ang=-180, type=DC, val="V"); +TwoTerminal R = resistor(V.beg, "R"); + +real lBar = 0.6*abs(R.end-V.end); +real hBar = 1mm; +pair cBar = (V.end+R.end)/2 - (0,R.len); + +// top, left, and right points for the left support +pair tLsup = cBar - (lBar/2, hBar/2); +pair lLsup = tLsup + supRadius * dir(-120); +pair rLsup = tLsup + supRadius * dir(-60); +wire(V.end, lLsup, udsq); +wire(lLsup, tLsup); +wire(tLsup, rLsup); + +// top, left, and right points for the right support +pair tRsup = cBar + (lBar/2, -hBar/2); +pair lRsup = tRsup + supRadius * dir(-120); +pair rRsup = tRsup + supRadius * dir(-60); +wire(R.end, rRsup, udsq); +wire(rRsup, tRsup); +wire(tRsup, lRsup); + +Vector Bs[]; +real dBy = supRadius*2/sqrt(3); +int i; +Bs.push(BField(cBar-(0,dBy), phi=-90)); +Bs.push(BField(cBar+(0,dBy), phi=-90, "\vect{B}")); +Bs.push(BField(cBar+(-lBar/2,dBy), phi=-90)); +Bs.push(BField(cBar+(+lBar/2,dBy), phi=-90)); +for (i=0; i0$ is moving at speed $v$ in the +$+z$-direction through a region of uniform magnetic field \vect{B}. +The magnetic force on the particle is $\vect{F}=F_0(3\ihat+4\jhat)$, +where $F_0$ is a positive constant. \Part{a} Determine the components +$B_x$, $B_y$, and $B_z$, or at least as many of the three components +as is possible from the information given. \Part{b} If it is given in +addition that the magnetic field has magnitude $6F_0/qv$, determine as +much as you can about the remaining components of \vect{B}. +\end{problem*} + +\begin{solution} +\end{solution} diff --git a/latex/problems/Young_and_Freedman_12/problem27.68.tex b/latex/problems/Young_and_Freedman_12/problem27.68.tex new file mode 100644 index 0000000..39201a3 --- /dev/null +++ b/latex/problems/Young_and_Freedman_12/problem27.68.tex @@ -0,0 +1,46 @@ +\begin{problem*}{27.68} +A $3.00\U{N}$ metal bar, $1.50\U{m}$ long and having a resistance of +$10.0\U{\Ohm}$, rests horizontally on conducting wires connecting it +to the circuit shown in Fig.~27.62. The bar is in a uniform, +horizontal, $1.60\U{T}$ magnnetic field and is not attached to the +wires in the circuit. What is the acceleration of the bar just after +the switch $S$ is closed? +\begin{center} +\begin{asy} +import Mechanics; +import ElectroMag; +import Circ; + +TwoTerminal Rv = resistor(ang=90, val="$10.0\U{\Ohm}$"); +TwoTerminal Rh = resistor(Rv.end, ang=-180, val="$25.0\U{\Ohm}$"); +TwoTerminal V = source(type=DC, "$120.0\U{V}$", draw=false); +centerto(Rv, V, Rh.len); V.draw(); +TwoTerminal S = switchSPST(Rv.end, type=NO, "$S$"); +wire(V.beg, (2*Rh.len, Rv.beg.y), udsq); +wire(V.end, Rh.end, udsq); +wire(S.end, (2*Rh.len, S.end.y), nsq); + +real xBar = (S.end.x+2*Rh.len)/2; + +Vector Bs[]; +int i, n = 4; +real dy = 1.4*Rv.len/(n-1); +real yBstart = Rv.mid.y - (n-1)*dy/2; +real xB = xBar - Rh.len/2; +for (i=0; i